K
Khách

Hãy nhập câu hỏi của bạn vào đây, nếu là tài khoản VIP, bạn sẽ được ưu tiên trả lời.

31 tháng 7 2017

F>=a^8/2(a^4+b^4)+b^8(b^4+c^4)+c^8/(c^4+a^4)>=(a^4+b^4+c^4)^2/4(a^4+b^4+c^4)=(a^4+b^4+c^4)/4

a^2+b^2+c^2>=ab+bc+ca=1.

3(a^4+b^4+c^4)>=(a^2+b^2+c^2)^2=1>>>a^4+b^4+c^4>=1/3

>>>F>=1/3/4=1/12

Dấu = xảy ra khi a=b=c(tự tính)

11 tháng 7 2017

tương tự Xem câu hỏi

25 tháng 9 2019

Ta có: \(P=\Sigma\frac{\left(\frac{1}{c^2}\right)}{\left(\frac{1}{a}+\frac{1}{b}\right)}\ge\frac{\left(\frac{1}{a}+\frac{1}{b}+\frac{1}{c}\right)^2}{2\left(\frac{1}{a}+\frac{1}{b}+\frac{1}{c}\right)}=\frac{\frac{1}{a}+\frac{1}{b}+\frac{1}{c}}{2}\ge\frac{\left(\frac{9}{a+b+c}\right)}{2}=\frac{3}{2}\)

Đẳng thức xảy ra khi a =b =c = 1.

True?

18 tháng 4 2020

Ta có : 

\(P=\frac{ab}{c^2\left(a+b\right)}+\frac{ac}{b^2\left(a+c\right)}+\frac{bc}{a^2\left(b+c\right)}\)

\(\Rightarrow P=\frac{\left(\frac{1}{c}\right)^2}{\frac{1}{a}+\frac{1}{b}}+\frac{\left(\frac{1}{b}\right)^2}{\frac{1}{c}+\frac{1}{a}}+\frac{\left(\frac{1}{a}\right)^2}{\frac{1}{c}+\frac{1}{b}}\)

\(\Rightarrow P\ge\frac{\left(\frac{1}{c}+\frac{1}{b}+\frac{1}{c}\right)^2}{\frac{1}{a}+\frac{1}{b}+\frac{1}{c}+\frac{1}{a}+\frac{1}{c}+\frac{1}{b}}\)

\(\Rightarrow P\ge\frac{\left(\frac{1}{a}+\frac{1}{b}+\frac{1}{c}\right)^2}{2\left(\frac{1}{a}+\frac{1}{b}+\frac{1}{c}\right)}\)

\(\Rightarrow P\ge\frac{1}{2}\left(\frac{1}{a}+\frac{1}{b}+\frac{1}{c}\right)\)

\(\Rightarrow P\ge\frac{1}{2}.\frac{9}{a+b+c}\)

\(\Rightarrow P\ge\frac{3}{2}\)

Dấu = xảy ra khi  a=b=c=1 

27 tháng 1 2020

Sử dụng trường hợp riêng của BĐT Schur. Với a,b,c là các sooa thực ko âm và k>0 ta luôn có :

\(a^k\left(a-b\right)\left(a-c\right)+b^k\left(b-c\right)\left(b-a\right)+c^k\left(c-a\right)\left(c-b\right)\ge0\)

27 tháng 1 2020

Anh tth_new ơi,mẹ em bắt em dirichlet ạ :( Mẹ em còn chỉ em bài toán tổng quát là:

Cho a,b,c dương,CMR:\(m\left(a^2+b^2+c^2\right)+abc+3m+2\ge\left(2m+1\right)\left(a+b+c\right)\)

\(BĐT\Leftrightarrow2\left(a^2+b^2+c^2\right)+abc+8\ge5\left(a+b+c\right)\)

 Thôi,đi vào giải quyết bài toán.

Trong 3 số \(a-1;b-1;c-1\) có ít nhất 2 số cùng dấu,giả sử đó là \(a-1;b-1\)

\(\Rightarrow\left(a-1\right)\left(b-1\right)\ge0\Rightarrow ab-a-b+1\ge0\Rightarrow abc\ge ac+bc-c\)

Khi đó BĐT tương đương với:

\(2\left(a^2+b^2+c^2\right)+abc+8\ge2\left(a^2+b^2+c^2\right)+ac+bc-c+8\)

Ta cần chứng minh:

\(2\left(a^2+b^2+c^2\right)+ac+bc-c+8\ge5\left(a+b+c\right)\)

\(\Leftrightarrow\left(b+c-2\right)^2+\left(c+a-2\right)^2+3\left(a-1\right)^2+3\left(b-1\right)^2+2\left(c-1\right)^2\ge0\) 

Hình như cái BĐT cuối đúng thì phải ạ.

Dấu "=" xảy ra tại a=b=c=1

NV
11 tháng 2 2020

Mới nghĩ ra 3 câu:

a/ \(\frac{ab}{\sqrt{\left(1-c\right)^2\left(1+c\right)}}=\frac{ab}{\sqrt{\left(a+b\right)^2\left(1+c\right)}}\le\frac{ab}{2\sqrt{ab\left(1+c\right)}}=\frac{1}{2}\sqrt{\frac{ab}{1+c}}\)

\(\sum\sqrt{\frac{ab}{1+c}}\le\sqrt{2\sum\frac{ab}{1+c}}\)

\(\sum\frac{ab}{1+c}=\sum\frac{ab}{a+c+b+c}\le\frac{1}{4}\sum\left(\frac{ab}{a+c}+\frac{ab}{b+c}\right)=\frac{1}{4}\)

c/ \(ab+bc+ca=2abc\Rightarrow\frac{1}{a}+\frac{1}{b}+\frac{1}{c}=2\)

Đặt \(\left(x;y;z\right)=\left(\frac{1}{a};\frac{1}{b};\frac{1}{c}\right)\Rightarrow x+y+z=2\)

\(VT=\sum\frac{x^3}{\left(2-x\right)^2}\)

Ta có đánh giá: \(\frac{x^3}{\left(2-x\right)^2}\ge x-\frac{1}{2}\) \(\forall x\in\left(0;2\right)\)

\(\Leftrightarrow2x^3\ge\left(2x-1\right)\left(x^2-4x+4\right)\)

\(\Leftrightarrow9x^2-12x+4\ge0\Leftrightarrow\left(3x-2\right)^2\ge0\)

d/ Ta có đánh giá: \(\frac{x^4+y^4}{x^3+y^3}\ge\frac{x+y}{2}\)

\(\Leftrightarrow\left(x-y\right)^2\left(x^2+xy+y^2\right)\ge0\)

11 tháng 2 2020

Akai Haruma, Nguyễn Ngọc Lộc , @tth_new, @Băng Băng 2k6, @Trần Thanh Phương, @Nguyễn Việt Lâm

Mn giúp e vs ạ! Thanks!

22 tháng 3 2016

B=1^8trên1^2

22 tháng 3 2016

\(\frac{1}{12}\)

AH
Akai Haruma
Giáo viên
2 tháng 3 2017

Bài 3)

BĐT cần chứng minh tương đương với:

\(\left ( \frac{a}{a+b} \right )^2+\left ( \frac{b}{b+c} \right )^2+\left ( \frac{c}{c+a} \right )^2\geq \frac{1}{2}\left ( 3-\frac{a}{a+b}-\frac{b}{b+c}-\frac{c}{c+a} \right )\)

Để cho gọn, đặt \((x,y,z)=\left (\frac{b}{a},\frac{c}{b},\frac{a}{c}\right)\) \(\Rightarrow xyz=1\).

BĐT được viết lại như sau:

\(A=2\left [ \frac{1}{(x+1)^2}+\frac{1}{(y+1)^2}+\frac{1}{(z+1)^2} \right ]+\frac{1}{x+1}+\frac{1}{y+1}+\frac{1}{z+1}\geq 3\) \((\star)\)

Ta nhớ đến hai bổ đề khá quen thuộc sau:

Bổ đề 1: Với \(a,b>0\) thì \(\frac{1}{(a+1)^2}+\frac{1}{(b+1)^2}\geq \frac{1}{ab+1}\)

Cách CM rất đơn giản, Cauchy - Schwarz:

\((a+1)^2\leq (a+b)(a+\frac{1}{b})\Rightarrow \frac{1}{(a+1)^2}\geq \frac{b}{(a+b)(ab+1)}\)

Tương tự với biểu thức còn lại và cộng vào thu được đpcm

Bổ đề 2: Với \(x,y>0,xy\geq 1\) thì \(\frac{1}{x^2+1}+\frac{1}{y^2+1}\geq \frac{2}{xy+1}\)

Cách CM: Quy đồng ta có đpcm.

Do tính hoán vị nên không mất tổng quát giả sử \(z=\min (x,y,z)\)

\(\Rightarrow xy\geq 1\). Áp dụng hai bổ đề trên:

\(A\geq 2\left [ \frac{1}{xy+1}+\frac{1}{(z+1)^2} \right ]+\frac{2}{\sqrt{xy}+1}+\frac{1}{z+1}=2\left [ \frac{z}{z+1}+\frac{1}{(z+1)^2} \right ]+\frac{2\sqrt{z}}{\sqrt{z}+1}+\frac{1}{z+1}\)

\(\Leftrightarrow A\geq \frac{2(z^2+z+1)}{(z+1)^2}+\frac{1}{z+1}+2-\frac{2}{\sqrt{z}+1}\geq 3\)

\(\Leftrightarrow 2\left [ \frac{z^2+z+1}{(z+1)^2}-\frac{3}{4} \right ]+\frac{1}{z+1}-\frac{1}{2}-\left ( \frac{2}{\sqrt{z}+1}-1 \right )\geq 0\)

\(\Leftrightarrow \frac{(z-1)^2}{2(z+1)^2}-\frac{z-1}{2(z+1)}+\frac{z-1}{(\sqrt{z}+1)^2}\geq 0\Leftrightarrow (z-1)\left [ \frac{1}{(\sqrt{z}+1)^2}-\frac{1}{(z+1)^2} \right ]\geq 0\)

\(\Leftrightarrow \frac{\sqrt{z}(\sqrt{z}-1)^2(\sqrt{z}+1)(z+\sqrt{z}+2)}{(\sqrt{z}+1)^2(z+1)^2}\geq 0\) ( luôn đúng với mọi \(z>0\) )

Do đó \((\star)\) được cm. Bài toán hoàn tất.

Dấu bằng xảy ra khi \(a=b=c\)

P/s: Nghỉ tuyển lâu rồi giờ mới gặp mấy bài BĐT phải động não. Khuya rồi nên xin phép làm bài 3 trước. Hai bài kia xin khiếu. Nếu làm đc chắc tối mai sẽ post.

2 tháng 3 2017

Bài 1:

Cho \(a=b=c=\dfrac{1}{\sqrt{3}}\). Khi đó \(M=\sqrt{3}-2\)

Ta sẽ chứng minh nó là giá trị nhỏ nhất

Thật vậy, đặt c là giá trị nhỏ nhất của a,b,c. Khi đó, ta cần chứng minh

\(\frac{a^2}{b}+\frac{b^2}{c}+\frac{c^2}{a}-\frac{2(a^2+b^2+c^2)}{\sqrt{ab+ac+bc}}\geq(\sqrt3-2)\sqrt{ab+ac+bc}\)

\(\Leftrightarrow\sqrt{ab+ac+bc}\left(\frac{a^2}{b}+\frac{b^2}{c}+\frac{c^2}{a}-\sqrt{3(ab+ac+bc)}\right)\geq2(a^2+b^2+c^2-ab-ac-bc)\)

\(\Leftrightarrow\frac{a^2}{b}+\frac{b^2}{a}-a-b+\frac{b^2}{c}+\frac{c^2}{a}-\frac{b^2}{a}-c+a+b+c-\sqrt{3(ab+ac+bc)}\geq\)

\(\geq2((a-b)^2+(c-a)(c-b))\)

\(\Leftrightarrow(a-b)^2\left(\frac{1}{a}+\frac{1}{b}-2\right)+(c-a)(c-b)\left(\frac{1}{a}+\frac{b}{ac}-2\right)+a+b+c-\sqrt{3(ab+ac+bc)}\geq0\)

Đúng bởi \(\frac{1}{a}+\frac{1}{b}-2>0;\frac{1}{a}+\frac{b}{ac}-2\geq\frac{1}{a}+\frac{1}{a}-2>0\)

\(a+b+c-\sqrt{3(ab+ac+bc)}=\frac{(a-b)^2+(c-a)(c-b)}{a+b+c+\sqrt{3(ab+ac+bc)}}\geq0\)

BĐT đã được c/m. Vậy \(M_{Min}=\sqrt{3}-2\Leftrightarrow a=b=c=\dfrac{1}{\sqrt{3}}\)

P/s: Nhìn qua thấy ngon mà làm mới thấy thật sự là "choáng"